LSAT and Law School Admissions Forum

Get expert LSAT preparation and law school admissions advice from PowerScore Test Preparation.

User avatar
 Dave Killoran
PowerScore Staff
  • PowerScore Staff
  • Posts: 5853
  • Joined: Mar 25, 2011
|
#87491
Complete Question Explanation
(The complete setup for this game can be found here: lsat/viewtopic.php?f=150&t=2097)

The correct answer choice is (A)

If F volunteers, then a contrapositive of the long chain inference occurs:

G2-Q11-d1.png

The only two variables unaccounted for are V and S, and neither is affected by any of the conditions in the chain above. Thus, V and S can separately volunteer or not volunteer, although at least one of the two must volunteer per the condition in the third rule.

Answer choice (A) is correct because L must volunteer.

Answer choice (B) can be eliminated because S does not have to volunteer.

Answer choice (A) can be eliminated because V does not have to volunteer.

Answer choice (D) can be eliminated because it is possible for four or five employees to volunteer.

Answer choice (E) can be eliminated because it is possible for three or five employees to volunteer.
You do not have the required permissions to view the files attached to this post.

Get the most out of your LSAT Prep Plus subscription.

Analyze and track your performance with our Testing and Analytics Package.